MD2 Flashcards

1
Q

When admitting an acutely confused patient with a head injury, which action should the nurse take?

a. Ask family members about the patient’s health history.
b. Ask leading questions to assist in obtaining health data.
c. Wait until the patient is better oriented to ask questions.
d. Obtain only the physiologic neurologic assessment data.

A

ANS: A
When admitting a patient who is likely to be a poor historian, the nurse should obtain health history information from others who have knowledge about the patient’s health. Waiting until the patient is oriented or obtaining only physiologic data will result in incomplete assessment data, which could adversely affect decision making about treatment. Asking leading questions may result in inaccurate or incomplete information.
DIF: Cognitive Level: Apply (application) REF: 1301 TOP: Nursing Process: Assessment MSC: NCLEX: Physiological Integrity

How well did you know this?
1
Not at all
2
3
4
5
Perfectly
2
Q

The nurse performing a focused assessment of left posterior temporal lobe functions will assess the patient for

a. sensation on the left side of the body.
b. reasoning and problem-solving ability.
c. ability to understand written and oral language.
d. voluntary movements on the right side of the body.

A

ANS: C
The posterior temporal lobe integrates the visual and auditory input for language comprehension. Reasoning and problem solving are functions of the anterior frontal lobe. Sensation on the left side of the body is located in the right postcentral gyrus. Voluntary movement on the right side is controlled in the left precentral gyrus.
DIF: Cognitive Level: Apply (application) REF: 1298 TOP: Nursing Process: Assessment MSC: NCLEX: Physiological Integrity

How well did you know this?
1
Not at all
2
3
4
5
Perfectly
3
Q

Which finding would the nurse expect when assessing the legs of a patient who has a lower motor neuron lesion?

A. Spasticity
B. Flaccidity
C.Impaired sensation
D. Hyperactive reflexes

A

ANS: B
Because the cell bodies of lower motor neurons are located in the spinal cord, damage to the neuron will decrease motor activity of the affected muscles. Spasticity and hyperactive reflexes are caused by upper motor neuron damage. Sensation is not impacted by motor neuron lesions.
DIF: Cognitive Level: Understand (comprehension) REF: 1296 TOP: Nursing Process: Assessment MSC: NCLEX: Physiological Integrity

How well did you know this?
1
Not at all
2
3
4
5
Perfectly
4
Q
Propranolol (Inderal), a β-adrenergic blocker that inhibits sympathetic nervous system activity, is prescribed for a patient who has extreme anxiety about public speaking. The nurse monitors the patient for
A dry mouth.
B bradycardia.
C. Constipation
D. urinary retention.
A

ANS: B
Inhibition of the fight-or-flight response leads to a decreased heart rate. Dry mouth, constipation, and urinary retention are associated with peripheral nervous system blockade.

How well did you know this?
1
Not at all
2
3
4
5
Perfectly
5
Q

To assess the functions of the trigeminal and facial nerves (CNs V and VII), the nurse should

a. check for unilateral eyelid droop.
b. shine a light into the patient’s pupil.
c. touch a cotton wisp strand to the cornea.
d. have the patient read a magazine or book.

A

ANS: C
The trigeminal and facial nerves are responsible for the corneal reflex. The optic nerve is tested by having the patient read a Snellen chart or a newspaper. Assessment of pupil response to light and ptosis are used to evaluate function of the oculomotor nerve.

How well did you know this?
1
Not at all
2
3
4
5
Perfectly
6
Q

Which action will the nurse include in the plan of care for a patient with impaired functioning of the left glossopharyngeal nerve (CN IX) and vagus nerve (CN X)?

A. Assist to stand and ambulate.
B. Withhold oral fluids and food.
C. Insert an oropharyngeal airway.
D. Apply artificial tears every hour.

A

ANS: B
The glossopharyngeal and vagus nerves innervate the pharynx and control the gag reflex. A patient with impaired function of these nerves is at risk for aspiration. An oral airway may be needed when a patient is unconscious and unable to maintain the airway, but it will not decrease aspiration risk. Taste and eye blink are controlled by the facial nerve. Balance and coordination are cerebellar functions.

How well did you know this?
1
Not at all
2
3
4
5
Perfectly
7
Q

An unconscious male patient has just arrived in the emergency department with a head injury caused by a motorcycle crash. Which order should the nurse question?

a. Obtain x-rays of the skull and spine.
b. Prepare the patient for lumbar puncture.
c. Send for computed tomography (CT) scan.
d. Perform neurologic checks every 15 minutes.

A

ANS: B
After a head injury, the patient may be experiencing intracranial bleeding and increased intracranial pressure, and herniation of the brain could result if lumbar puncture is performed. The other orders are appropriate.

How well did you know this?
1
Not at all
2
3
4
5
Perfectly
8
Q

A patient with suspected meningitis is scheduled for a lumbar puncture. Before the procedure, the nurse will plan to

a. enforce NPO status for 4 hours.
b. transfer the patient to radiology.
c. administer a sedative medication.
d. help the patient to a lateral position

A

ANS: D
For a lumbar puncture, the patient lies in the lateral recumbent position. The procedure does not usually require a sedative, is done in the patient room, and has no risk for aspiration.

How well did you know this?
1
Not at all
2
3
4
5
Perfectly
9
Q
During the neurologic assessment, the patient is unable to respond verbally to the nurse but cooperates with the nurse’s directions to move his hands and feet. The nurse will suspect
A. cerebellar injury. 
B. a brainstem lesion. 
C. Frontal loba damage 
D. a temporal lobe lesion.
A
ANS: C
Expressive speech (ability to express the self in language) is controlled by Broca’s area in the frontal lobe. The temporal lobe contains Wernicke’s area, which is responsible for receptive speech (ability to understand language input). The cerebellum and brainstem do not affect higher cognitive functions such as speech.
How well did you know this?
1
Not at all
2
3
4
5
Perfectly
10
Q
A patient has a tumor in the cerebellum. The nurse will plan interventions to
A. prevent falls. 
B. stabilize mood. 
C. Avoid aspiration
D. improve memory.
A

ANS: A
Because functions of the cerebellum include coordination and balance, the patient with dysfunction is at risk for falls. The cerebellum does not affect memory, mood, or swallowing ability.

How well did you know this?
1
Not at all
2
3
4
5
Perfectly
11
Q

Which problem can the nurse expect for a patient who has a positive Romberg test result?

A. Pain
B. Falls
C. Aphasia
D. Confusion

A

ANS: B
A positive Romberg test result indicates that the patient has difficulty maintaining balance when standing with the eyes closed. The Romberg does not test for orientation, thermoregulation, or discomfort.

How well did you know this?
1
Not at all
2
3
4
5
Perfectly
12
Q

The nurse will anticipate teaching a patient with a possible seizure disorder about which test?

A. Cerebral angiography
B. Evoked potential studies
C. Electromyography (EMG)
D. Electroencephalography (EEG)

A

ANS: D
Seizure disorders are usually assessed using EEG testing. Evoked potential is used to diagnose problems with the visual or auditory systems. Cerebral angiography is used to diagnose vascular problems. EMG is used to evaluate electrical innervation to skeletal muscle.

How well did you know this?
1
Not at all
2
3
4
5
Perfectly
13
Q

Which nursing action will be included in the plan of care for a patient who has had cerebral angiography?

a. Monitor for headache and photophobia.
b. Keep patient NPO until gag reflex returns.
c. Check pulse and blood pressure frequently.
d. Assess orientation to person, place, and time.

A

ANS: C
Because a catheter is inserted into an artery (e.g., the femoral artery) during cerebral angiography, the nurse should assess for bleeding after this procedure that can affect pulse and blood pressure. The other nursing assessments are not needed after angiography

How well did you know this?
1
Not at all
2
3
4
5
Perfectly
14
Q
Which equipment will the nurse obtain to assess vibration sense in a patient with diabetes who has peripheral nerve dysfunction?
A. Sharp pin 
B. Tuning fork 
C. Reflex hammer
D. Calibrated compass
A

ANS: B
Vibration sense is testing by touching the patient with a vibrating tuning fork. The other equipment is needed for testing of pain sensation, reflexes, and two-point discrimination.

How well did you know this?
1
Not at all
2
3
4
5
Perfectly
15
Q

Which information about a 76-yr-old patient should the nurse report as uncharacteristic of normal aging?

a. Triceps reflex response graded at 1/5
b. Unintended weight loss of 15 pounds
c. 10 mm Hg orthostatic drop in systolic blood pressure
d. Patient complaint of chronic difficulty in falling asleep

A

ANS: B
Although changes in appetite are normal with aging, a 15-lb weight loss requires further investigation. Orthostatic drops in blood pressure, changes in sleep patterns, and slowing of reflexes are normal changes in aging.

How well did you know this?
1
Not at all
2
3
4
5
Perfectly
16
Q

The charge nurse is observing a new staff nurse who is assessing a patient with a traumatic spinal cord injury for sensation. Which action indicates a need for further teaching of the new nurse about neurologic assessment?

a. The new nurse tests for light touch before testing for pain.
b. The new nurse has the patient close the eyes during testing.
c. The new nurse asks the patient if the instrument feels sharp.
d. The new nurse uses an irregular pattern to test for intact touch

A

ANS: C
When performing a sensory assessment, the nurse should not provide verbal clues. The other actions by the new nurse are appropriate.

How well did you know this?
1
Not at all
2
3
4
5
Perfectly
17
Q

Which cerebrospinal fluid analysis result should the nurse recognize as abnormal and communicate to the health care provider?

a. Specific gravity of 1.007
b. Protein of 65 mg/dL (0.65 g/L)
c. Glucose of 45 mg/dL (1.7 mmol/L)
d. White blood cell (WBC) count of 4 cells/μL

A

ANS: B

The protein level is high. The specific gravity, WBCs, and glucose values are normal.

How well did you know this?
1
Not at all
2
3
4
5
Perfectly
18
Q

A 39-yr-old patient with a suspected herniated intervertebral disc is scheduled for a myelogram. Which information communicated by the nurse to the health care provider before the procedure would change the procedural plans?

a. The patient is anxious about the test results.
b. The patient reports a previous allergy to shellfish.
c. The patient has back pain when lying flat for more than 4 hours.
d. The patient drank apple juice 4 hours before the scheduled procedure.

A

ANS: B
A contrast medium containing iodine is injected into the subarachnoid space during a myelogram. The patient’s allergy would contraindicate the use of this medium. The health care provider may need to modify the orders to prevent back pain, but this can be done after the procedure. Clear liquids are usually considered safe up to 4 hours before a diagnostic or surgical procedure. The patient’s anxiety should be addressed, but procedural plans would not need to be changed.

How well did you know this?
1
Not at all
2
3
4
5
Perfectly
19
Q
The priority nursing assessment for a patient being admitted with a brainstem infarction is
A. pupil reaction.
B. respiratory rate. 
C. Reflex reaction time
D. level of consciousness
A

ANS: B
Vital centers that control respiration are located in the medulla and part of the brainstem, and will require priority assessments because changes in respiratory function may be life threatening. The other information will also be obtained by the nurse but is not as urgent.

How well did you know this?
1
Not at all
2
3
4
5
Perfectly
20
Q

Several patients have been hospitalized for diagnosis of neurologic problems. Which patient will the nurse assess first?
a. A patient with a transient ischemic attack (TIA) returning from carotid duplex
studies
b. A patient with a brain tumor who has just arrived on the unit after a cerebral
angiogram
c. A patient with a seizure disorder who has just completed an
electroencephalogram (EEG)
d. A patient prepared for a lumbar puncture whose health care provider is waiting
for assistance

A

ANS: B
Because cerebral angiograms require insertion of a catheter into the femoral artery, bleeding is a possible complication. The nurse will need to check the pulse, blood pressure, and the catheter insertion site in the groin as soon as the patient arrives. Carotid duplex studies and EEG are noninvasive. The nurse will need to assist with the lumbar puncture as soon as possible, but monitoring for hemorrhage after cerebral angiogram has a higher priority.

How well did you know this?
1
Not at all
2
3
4
5
Perfectly
21
Q

Which assessments will the nurse make to monitor a patient’s cerebellar function (select all that apply)?

a. Test for graphesthesia.
b. Observe arm swing with gait.
c. Perform the finger-to-nose test.
d. Assess heat and cold sensation.
e. Measure strength against resistance.

A

ANS: B,C
The cerebellum is responsible for coordination and is assessed by looking at the patient’s gait and the finger-to-nose test. The other assessments will be used for other parts of the neurologic assessment.

How well did you know this?
1
Not at all
2
3
4
5
Perfectly
22
Q

Family members of a patient who has a traumatic brain injury ask the nurse about the purpose of the ventriculostomy system being used for intracranial pressure monitoring. Which response by the nurse is best for this situation?

a. “This type of monitoring system is complex and it is managed by skilled staff.”
b. “The monitoring system helps show whether blood flow to the brain is adequate.”
c. “The ventriculostomy monitoring system helps check for alterations in cerebral perfusion pressure.”
d. “This monitoring system has multiple benefits including facilitation of cerebrospinal fluid drainage.”

A

ANS: B
Short and simple explanations should be given initially to patients and family members. The other explanations are either too complicated to be easily understood or may increase the family members’ anxiety.

How well did you know this?
1
Not at all
2
3
4
5
Perfectly
23
Q

Admission vital signs for a brain-injured patient are blood pressure of 128/68 mm Hg, pulse of 110 beats/min, and of respirations 26 breaths/min. Which set of vital signs, if taken 1 hour later, will be of most concern to the nurse?

a. Blood pressure of 154/68 mm Hg, pulse of 56 beats/min, respirations of 12
breaths/min
b. Blood pressure of 134/72 mm Hg, pulse of 90 beats/min, respirations of 32
breaths/min
c. Blood pressure of 148/78 mm Hg, pulse of 112 beats/min, respirations of 28
breaths/min
d. Blood pressure of 110/70 mm Hg, pulse of 120 beats/min, respirations of 30
breaths/min

A

ANS: A
Systolic hypertension with widening pulse pressure, bradycardia, and respiratory changes represent Cushing’s triad. These findings indicate that the intracranial pressure (ICP) has increased, and brain herniation may be imminent unless immediate action is taken to reduce ICP. The other vital signs may indicate the need for changes in treatment, but they are not indicative of an immediately life-threatening process

How well did you know this?
1
Not at all
2
3
4
5
Perfectly
24
Q
When a brain-injured patient responds to nail bed pressure with internal rotation, adduction, and flexion of the arms, the nurse reports the response as
A. Flexion withdrawal. 
B. Localization of pain.
C. Decorticate posturing.
D. Decerebrate posturing.
A

ANS: C
Internal rotation, adduction, and flexion of the arms in an unconscious patient is documented as decorticate posturing. Extension of the arms and legs is decerebrate posturing. Because the flexion is generalized, it does not indicate localization of pain or flexion withdrawal.

How well did you know this?
1
Not at all
2
3
4
5
Perfectly
25
Q
The nurse has administered prescribed IV mannitol (Osmitrol) to an unconscious patient. Which parameter should the nurse monitor to determine the medication’s effectiveness?
A. Blood pressure 
B. Oxygen saturation 
C. Intracranial pressure
D. Hemoglobin and hematocrit
A

ANS: C
Mannitol is an osmotic diuretic and will reduce cerebral edema and intracranial pressure. It may initially reduce hematocrit and increase blood pressure, but these are not the best parameters for evaluation of the effectiveness of the drug. O2 saturation will not directly improve as a result of mannitol administration.

How well did you know this?
1
Not at all
2
3
4
5
Perfectly
26
Q
A patient with a head injury opens his eyes to verbal stimulation, curses when stimulated, and does not respond to a verbal command to move but attempts to push away a painful stimulus. The nurse records the patient’s Glasgow Coma Scale score as 
A. 9. 
B. 11. 
C. 13
D. 15.
A

ANS: B

The patient has scores of 3 for eye opening, 3 for best verbal response, and 5 for best motor response.

How well did you know this?
1
Not at all
2
3
4
5
Perfectly
27
Q

An unconscious patient is admitted to the emergency department (ED) with a head injury. The patient’s spouse and teenage children stay at the patient’s side and ask many questions about the treatment being given. What action is best for the nurse to take?

a. Call the family’s pastor or spiritual advisor to take them to the chapel.
b. Ask the family to stay in the waiting room until the assessment is completed.
c. Allow the family to stay with the patient and briefly explain all procedures to
them.
d. Refer the family members to the hospital counseling service to deal with their
anxiety.

A

ANS: C
The need for information about the diagnosis and care is very high in family members of acutely ill patients. The nurse should allow the family to observe care and explain the procedures unless they interfere with emergent care needs. A pastor or counseling service can offer some support, but research supports information as being more effective. Asking the family to stay in the waiting room will increase their anxiety.

How well did you know this?
1
Not at all
2
3
4
5
Perfectly
28
Q

A patient who is unconscious has ineffective cerebral tissue perfusion and cerebral tissue swelling. Which nursing intervention will be included in the plan of care?

a. Encourage coughing and deep breathing.
b. Position the patient with knees and hips flexed.
c. Keep the head of the bed elevated to 30 degrees.
d. Cluster nursing interventions to provide rest periods.

A

ANS: C
The patient with increased intracranial pressure (ICP) should be maintained in the head-up position to help reduce ICP. Extreme flexion of the hips and knees increases abdominal pressure, which increases ICP. Because the stimulation associated with nursing interventions increases ICP, clustering interventions will progressively elevate ICP. Coughing increases intrathoracic p ressure and ICP.

How well did you know this?
1
Not at all
2
3
4
5
Perfectly
29
Q

A 20-yr-old male patient is admitted with a head injury after a collision while playing football. After noting that the patient has developed clear nasal drainage, which action should the nurse take?

a. Have the patient gently blow the nose.
b. Check the drainage for glucose content.
c. Teach the patient that rhinorrhea is expected after a head injury.
d. Obtain a specimen of the fluid to send for culture and sensitivity.

A

ANS: B
Clear nasal drainage in a patient with a head injury suggests a dural tear and cerebrospinal fluid (CSF) leakage. If the drainage is CSF, it will test positive for glucose. Fluid leaking from the nose will have normal nasal flora, so culture and sensitivity will not be useful. Blowing the nose is avoided to prevent CSF leakage.

How well did you know this?
1
Not at all
2
3
4
5
Perfectly
30
Q

Which action will the emergency department nurse anticipate for a patient diagnosed with a concussion who did not lose consciousness?

a. Coordinate the transfer of the patient to the operating room.
b. Provide discharge instructions about monitoring neurologic status.
c. Transport the patient to radiology for magnetic resonance imaging (MRI).
d. Arrange to admit the patient to the neurologic unit for 24 hours of observation.

A

ANS: B
A patient with a minor head trauma is usually discharged with instructions about neurologic monitoring and the need to return if neurologic status deteriorates. MRI, hospital admission, and surgery are not usually indicated in a patient with a concussion.

How well did you know this?
1
Not at all
2
3
4
5
Perfectly
31
Q

A patient who is suspected of having an epidural hematoma is admitted to the emergency department. Which action will the nurse expect to take?

a. Administer IV furosemide (Lasix).
b. Prepare the patient for craniotomy.
c. Initiate high-dose barbiturate therapy.
d. Type and crossmatch for blood transfusion.

A

ANS: B
The principal treatment for epidural hematoma is rapid surgery to remove the hematoma and prevent herniation. If intracranial pressure is elevated after surgery, furosemide or high-dose barbiturate therapy may be needed, but these will not be of benefit unless the hematoma is removed. Minimal blood loss occurs with head injuries, and transfusion is usually not necessary.

How well did you know this?
1
Not at all
2
3
4
5
Perfectly
32
Q

The nurse is admitting a patient with a basal skull fracture. The nurse notes ecchymoses around both eyes and clear drainage from the patient’s nose. Which admission order should the nurse question?

a. Keep the head of bed elevated.
b. Insert nasogastric tube to low suction.
c. Turn patient side to side every 2 hours.
d. Apply cold packs intermittently to face.

A

ANS: B
Rhinorrhea may indicate a dural tear with cerebrospinal fluid leakage. Insertion of a nasogastric tube will increase the risk for infections such as meningitis. Turning the patient, elevating the head, and applying cold packs are appropriate orders.

How well did you know this?
1
Not at all
2
3
4
5
Perfectly
33
Q
A college athlete is seen in the clinic 6 weeks after a concussion. Which assessment information will the nurse collect to determine whether the patient is developing postconcussion syndrome?
A. Short-term memory 
B. Muscle coordination 
C. Glasgow Coma Scale
D. Pupil reaction to light
A

ANS: A
Decreased short-term memory is one indication of postconcussion syndrome. The other data may be assessed but are not indications of postconcussion syndrome.

How well did you know this?
1
Not at all
2
3
4
5
Perfectly
34
Q

The nurse admitting a patient who has a right frontal lobe tumor would expect the patient may have

A. expressive aphasia.
B. impaired judgment.
C. Right-sided weakness
D. difficulty swallowing.

A

ANS: B
The frontal lobe controls intellectual activities such as judgment. Speech is controlled in the parietal lobe. Weakness and hemiplegia occur on the contralateral side from the tumor. Swallowing is controlled by the brainstem.

How well did you know this?
1
Not at all
2
3
4
5
Perfectly
35
Q

Which statement by patient who is being discharged from the emergency department (ED) after a concussion indicates a need for intervention by the nurse?

a. “I will return if I feel dizzy or nauseated.”
b. “I am going to drive home and go to bed.”
c. “I do not even remember being in an accident.”
d. “I can take acetaminophen (Tylenol) for my headache.”

A

ANS: B
After a head injury, the patient should avoid driving and operating heavy machinery. Retrograde amnesia is common after a concussion. The patient can take acetaminophen for headache and should return if symptoms of increased intracranial pressure such as dizziness or nausea occur.

How well did you know this?
1
Not at all
2
3
4
5
Perfectly
36
Q

After having a craniectomy and left anterior fossae incision, a 64-yr-old patient has impaired physical mobility related to decreased level of consciousness and weakness. An appropriate nursing intervention is to

a. cluster nursing activities to allow longer rest periods.
b. turn and reposition the patient side to side every 2 hours.
c. position the bed flat and log roll to reposition the patient.
d. perform range-of-motion (ROM) exercises every 4 hours.

A

ANS: D
ROM exercises will help prevent the complications of immobility. Patients with anterior craniotomies are positioned with the head elevated. The patient with a craniectomy should not be turned to the operative side. When the patient is weak, clustering nursing activities may lead to more fatigue and weakness.

How well did you know this?
1
Not at all
2
3
4
5
Perfectly
37
Q

A patient who has bacterial meningitis is disoriented and anxious. Which nursing action will be included in the plan of care?

a. Encourage family members to remain at the bedside.
b. Apply soft restraints to protect the patient from injury.
c. Keep the room well-lighted to improve patient orientation.
d. Minimize contact with the patient to decrease sensory input.

A

ANS: A
Patients with meningitis and disorientation will be calmed by the presence of someone familiar at the bedside. Restraints should be avoided because they increase agitation and anxiety. The patient requires frequent assessment for complications. The use of touch and a soothing voice will decrease anxiety for most patients. The patient will have photophobia, so the light should be dim.

How well did you know this?
1
Not at all
2
3
4
5
Perfectly
38
Q

The public health nurse is planning a program to decrease the incidence of meningitis in teenagers and young adults. Which action is most likely to be effective?

a. Emphasize the importance of hand washing.
b. Immunize adolescents and college freshman.
c. Support serving healthy nutritional options in the college cafeteria.
d. Encourage adolescents and young adults to avoid crowds in the winter.

A

ANS: B
The Neisseria meningitides vaccination is recommended for children ages 11 and 12 years, unvaccinated teens entering high school, and college freshmen. Hand washing may help decrease the spread of bacteria, and good nutrition may increase resistance to infection. but those are not as effective as immunization. Because adolescents and young adults are in school or the workplace, avoiding crowds is not realistic.

How well did you know this?
1
Not at all
2
3
4
5
Perfectly
39
Q

A patient has been admitted with meningococcal meningitis. Which observation by the nurse requires action?

a. The patient receives a regular diet tray.
b. The bedrails on both sides of the bed are elevated.
c. Staff have turned off the lights in the patient’s room.
d. Staff have entered the patient’s room without a mask.

A

ANS: D
Meningococcal meningitis is spread by respiratory secretions, so it is important to maintain respiratory isolation as well as standard precautions. Because the patient may be confused and weak, bedrails should be elevated at both the foot and head of the bed. Low light levels in the room decrease pain caused by photophobia. Nutrition is an important aspect of care in a patient with meningitis.

How well did you know this?
1
Not at all
2
3
4
5
Perfectly
40
Q

When assessing a 53-yr-old patient with bacterial meningitis, the nurse obtains the following data. Which finding requires the most immediate intervention?

a. The patient exhibits nuchal rigidity.
b. The patient has a positive Kernig’s sign.
c. The patient’s temperature is 101° F (38.3° C).
d. The patient’s blood pressure is 88/42 mm Hg.

A

ANS: D
Shock is a serious complication of meningitis, and the patient’s low blood pressure indicates the need for interventions such as fluids or vasopressors. Nuchal rigidity and a positive Kernig’s sign are expected with bacterial meningitis. The nurse should intervene to lower the temperature, but this is not as life threatening as the hypotension.

How well did you know this?
1
Not at all
2
3
4
5
Perfectly
41
Q

A patient admitted with a diffuse axonal injury has a systemic blood pressure (BP) of 106/52 mm Hg and an intracranial pressure (ICP) of 14 mm Hg. Which action should the nurse take first?

a. Document the BP and ICP in the patient’s record.
b. Report the BP and ICP to the health care provider.
c. Elevate the head of the patient’s bed to 60 degrees.
d. Continue to monitor the patient’s vital signs and ICP.

A

ANS: B
Calculate the cerebral perfusion pressure (CPP): (CPP = Mean arterial pressure [MAP] – ICP). MAP = DBP + 1/3 (Systolic blood pressure [SBP] – Diastolic blood pressure [DBP]). Therefore the MAP is 70, and the CPP is 56 mm Hg, which are below the normal values of 60 to 100 mm Hg and are approaching the level of ischemia and neuronal death. Immediate changes in the patient’s therapy such as fluid infusion or vasopressor administration are needed to improve the CPP. Adjustments in the head elevation should only be done after consulting with the health care provider. Continued monitoring and documentation will also be done, but they are not the first actions that the nurse should take.

How well did you know this?
1
Not at all
2
3
4
5
Perfectly
42
Q

After endotracheal suctioning, the nurse notes that the intracranial pressure (ICP) for a patient with a traumatic head injury has increased from 14 to 17 mm Hg. Which action should the nurse take first?

a. Document the increase in intracranial pressure.
b. Ensure that the patient’s neck is in neutral position.
c. Notify the health care provider about the change in pressure.
d. Increase the rate of the prescribed propofol (Diprivan) infusion.

A

ANS: B
Because suctioning will cause a transient increase in ICP, the nurse should initially check for other factors that might be contributing to the increase and observe the patient for a few minutes. Documentation is needed, but this is not the first action. There is no need to notify the health care provider about this expected reaction to suctioning. Propofol is used to control patient anxiety or agitation. There is no indication that anxiety has contributed to the increase in ICP.

How well did you know this?
1
Not at all
2
3
4
5
Perfectly
43
Q

Which patient is most appropriate for the intensive care unit (ICU) charge nurse to assign to a registered nurse (RN) who has floated from the medical unit?

a. A 45-yr-old patient receiving IV antibiotics for meningococcal meningitis
b. A 35-yr-old patient with intracranial pressure (ICP) monitoring after a head injury
c. A 25-yr-old patient admitted with a skull fracture and craniotomy the previous day
d. A 55-yr-old patient who has increased intracranial pressure (ICP) and is receiving hyperventilation therapy

A

ANS: A
An RN who works on a medical unit will be familiar with administration of IV antibiotics and with meningitis. The patient recovering from a craniotomy, the patient with an ICP monitor, and the patient on a ventilator should be assigned to an RN familiar with the care of critically ill patients.

How well did you know this?
1
Not at all
2
3
4
5
Perfectly
44
Q

A male patient who has possible cerebral edema has a serum sodium level of 116 mEq/L (116 mmol/L) and a decreasing level of consciousness (LOC). He is now complaining of a headache. Which prescribed interventions should the nurse implement first?

a. Administer IV 5% hypertonic saline.
b. Draw blood for arterial blood gases (ABGs).
c. Send patient for computed tomography (CT).
d. Administer acetaminophen (Tylenol) 650 mg orally.

A

ANS: A
The patient’s low sodium indicates that hyponatremia may be causing the cerebral edema. The nurse’s first action should be to correct the low sodium level. Acetaminophen (Tylenol) will have minimal effect on the headache because it is caused by cerebral edema and increased intracranial pressure (ICP). Drawing ABGs and obtaining a CT scan may provide some useful information, but the low sodium level may lead to seizures unless it is addressed quickly.

How well did you know this?
1
Not at all
2
3
4
5
Perfectly
45
Q

After the emergency department nurse has received a status report on the following patients who have been admitted with head injuries, which patient should the nurse assess first?

a. A 20-yr-old patient whose cranial x-ray shows a linear skull fracture
b. A 50-yr-old patient who has an initial Glasgow Coma Scale score of 13
c. A 30-yr-old patient who lost consciousness for a few seconds after a fall
d. A 40-yr-old patient whose right pupil is 10 mm and unresponsive to light

A

ANS: D
The dilated and nonresponsive pupil may indicate an intracerebral hemorrhage and increased intracranial pressure. The other patients are not at immediate risk for complications such as herniation.

How well did you know this?
1
Not at all
2
3
4
5
Perfectly
46
Q

The nurse is caring for a patient who was admitted the previous day with a basilar skull fracture after a motor vehicle crash. Which assessment finding indicates a possible complication that should be reported to the health care provider?

a. Complaint of severe headache
b. Large contusion behind left ear
c. Bilateral periorbital ecchymosis
d. Temperature of 101.4° F (38.6° C)

A

ANS: D
Patients who have basilar skull fractures are at risk for meningitis, so the elevated temperature should be reported to the health care provider. The other findings are typical of a patient with a basilar skull fracture.

How well did you know this?
1
Not at all
2
3
4
5
Perfectly
47
Q

After evacuation of an epidural hematoma, a patient’s intracranial pressure (ICP) is being monitored with an intraventricular catheter. Which information obtained by the nurse requires urgent communication with the health care provider?

a. Pulse of 102 beats/min
b. Temperature of 101.6° F
c. Intracranial pressure of 15 mm Hg
d. Mean arterial pressure of 90 mm Hg

A

ANS: B
Infection is a serious consideration with ICP monitoring, especially with intraventricular catheters. The temperature indicates the need for antibiotics or removal of the monitor. The ICP, arterial pressure, and apical pulse only require ongoing monitoring at this time.

How well did you know this?
1
Not at all
2
3
4
5
Perfectly
48
Q

The charge nurse observes an inexperienced staff nurse caring for a patient who has had a craniotomy for resection of a brain tumor. Which action by the inexperienced nurse requires the charge nurse to intervene?

a. The staff nurse assesses neurologic status every hour.
b. The staff nurse elevates the head of the bed to 30 degrees.
c. The staff nurse suctions the patient routinely every 2 hours.
d. The staff nurse administers an analgesic before turning the patient.

A

ANS: C
Suctioning increases intracranial pressure and should only be done when the patient’s respiratory condition indicates it is needed. The other actions by the staff nurse are appropriate.

How well did you know this?
1
Not at all
2
3
4
5
Perfectly
49
Q

A 68-yr-old male patient is brought to the emergency department (ED) by ambulance after being found unconscious on the bathroom floor by his spouse. Which action will the nurse take first?

a. Check oxygen saturation.
b. Assess pupil reaction to light.
c. Palpate the head for injuries
d. Verify Glasgow Coma Scale (GCS) score.

A

ANS: A
Airway patency and breathing are the most vital functions and should be assessed first. The neurologic assessments should be accomplished next and additional assessment after that.

How well did you know this?
1
Not at all
2
3
4
5
Perfectly
50
Q

. A patient with increased intracranial pressure after a head injury has a ventriculostomy in place. Which action can the nurse delegate to unlicensed assistive personnel (UAP) who regularly work in the intensive care unit?

a. Document intracranial pressure every hour.
b. Turn and reposition the patient every 2 hours.
c. Check capillary blood glucose level every 6 hours.
d. Monitor cerebrospinal fluid color and volume hourly.

A

ANS: C
Experienced UAP can obtain capillary blood glucose levels when they have been trained and evaluated in the skill. Monitoring and documentation of cerebrospinal fluid (CSF) color and intracranial pressure (ICP) require registered nurse (RN)–level education and scope of practice. Although repositioning patients is frequently delegated to UAP, repositioning a patient with a ventriculostomy is complex and should be supervised by the RN.

How well did you know this?
1
Not at all
2
3
4
5
Perfectly
51
Q

Which information about a 30-yr-old patient who is hospitalized after a traumatic brain injury requires the most rapid action by the nurse?

a. Intracranial pressure of 15 mm Hg
b. Cerebrospinal fluid (CSF) drainage of 25 mL/hr
c. Pressure of oxygen in brain tissue (PbtO2) is 14 mm Hg
d. Cardiac monitor shows sinus tachycardia at 120 beats/minute

A

ANS: C
The PbtO2 should be 20 to 40 mm Hg. Lower levels indicate brain ischemia. An intracranial pressure (ICP) of 15 mm Hg is at the upper limit of normal. CSF is produced at a rate of 20 to 30 mL/hr. The reason for the sinus tachycardia should be investigated, but the elevated heart rate is not as concerning as the decrease in PbtO2.

How well did you know this?
1
Not at all
2
3
4
5
Perfectly
52
Q

The nurse is caring for a patient who has a head injury and fractured right arm after being assaulted. Which assessment information requires rapid action by the nurse?

a. The apical pulse is slightly irregular.
b. The patient complains of a headache.
c. The patient is more difficult to arouse.
d. The blood pressure (BP) increases to 140/62 mm Hg.

A

ANS: C
The change in level of consciousness (LOC) is an indicator of increased intracranial pressure (ICP) and suggests that action by the nurse is needed to prevent complications. The change in BP should be monitored but is not an indicator of a need for immediate nursing action. Headache and a slightly irregular apical pulse are not unusual in a patient after a head injury.

How well did you know this?
1
Not at all
2
3
4
5
Perfectly
53
Q

The nurse is caring for a patient who has a head injury. Which finding, when reported to the health care provider, should the nurse expect will result in new prescribed interventions?

a. Pale yellow urine output of 1200 mL over the past 2 hours.
b. Ventriculostomy drained 40 mL of fluid in the past 2 hours.
c. Intracranial pressure spikes to 16 mm Hg when patient is turned.
d. LICOX brain tissue oxygenation catheter shows PbtO2 of 38 mm Hg.

A

ANS: A
The high urine output indicates that diabetes insipidus may be developing, and interventions to prevent dehydration need to be rapidly implemented. The other data do not indicate a need for any change in therapy.

How well did you know this?
1
Not at all
2
3
4
5
Perfectly
54
Q

While admitting a 42-yr-old patient with a possible brain injury after a car accident to the emergency department (ED), the nurse obtains the following information. Which finding is most important to report to the health care provider?

a. The patient takes warfarin (Coumadin) daily.
b. The patient’s blood pressure is 162/94 mm Hg.
c. The patient is unable to remember the accident.
d. The patient complains of a severe dull headache.

A

ANS: A
The use of anticoagulants increases the risk for intracranial hemorrhage and should be immediately reported. The other information would not be unusual in a patient with a head injury who had just arrived in the ED.

How well did you know this?
1
Not at all
2
3
4
5
Perfectly
55
Q

A patient being admitted with bacterial meningitis has a temperature of 102.5° F (39.2° C) and a severe headache. Which order should the nurse implement first?

a. Administer ceftizoxime (Cefizox) 1 g IV.
b. Give acetaminophen (Tylenol) 650 mg PO.
c. Use a cooling blanket to lower temperature.
d. Swab the nasopharyngeal mucosa for cultures.

A

ANS: D
Antibiotic therapy should be instituted rapidly in bacterial meningitis, but cultures must be done before antibiotics are started. As soon as the cultures are done, the antibiotic should be started. Hypothermia therapy and acetaminophen administration are appropriate but can be started after the other actions are implemented.

How well did you know this?
1
Not at all
2
3
4
5
Perfectly
56
Q

A patient with possible viral meningitis is admitted to the nursing unit after lumbar puncture was performed in the emergency department. Which action prescribed by the health care provider should the nurse question?

a. Restrict oral fluids to 1000 mL/day.
b. Elevate the head of the bed 20 degrees.
c. Administer ceftriaxone (Rocephin) 1 g IV every 12 hours.
d. Give ibuprofen (Motrin) 400 mg every 6 hours as needed for headache.

A

ANS: A
The patient with meningitis has increased fluid needs, so oral fluids should be encouraged. The other actions are appropriate. Slight elevation of the head of the bed will decrease headache without causing leakage of cerebrospinal fluid from the lumbar puncture site. Antibiotics should be administered until bacterial meningitis is ruled out by the cerebrospinal fluid analysis.

How well did you know this?
1
Not at all
2
3
4
5
Perfectly
57
Q

Which action will the public health nurse take to reduce the incidence of epidemic encephalitis in a community?

a. Teach about prophylactic antibiotics after exposure to encephalitis.
b. Encourage the use of effective insect repellent during mosquito season.
c. Remind patients that most cases of viral encephalitis can be cared for at home.
d. Arrange to screen school-age children for West Nile virus during the school year.

A

ANS: B
Epidemic encephalitis is usually spread by mosquitoes and ticks. Use of insect repellent is effective in reducing risk. Encephalitis frequently requires that the patient be hospitalized in an intensive care unit during the initial stages. Antibiotic pro phylaxis is not used to prevent encephalitis because most encephalitis is viral. West Nile virus is most common in adults over age 50 during the summer and early fall.

How well did you know this?
1
Not at all
2
3
4
5
Perfectly
58
Q

Which question will the nurse ask a patient who has been admitted with a benign occipital lobe tumor to assess for functional deficits?

a. “Do you have difficulty in hearing?”
b. “Are you experiencing visual problems?”
c. “Are you having any trouble with your balance?”
d. “Have you developed any weakness on one side?”

A

ANS: B
Because the occipital lobe is responsible for visual reception, the patient with a tumor in this area is likely to have problems with vision. The other questions will be better for assessing function of the temporal lobe, cerebellum, and frontal lobe.

How well did you know this?
1
Not at all
2
3
4
5
Perfectly
59
Q

56(38) During change-of-shift report, the nurse learns that a patient with a head injury has decorticate posturing to noxious stimulation. Which positioning shown in the accompanying figure will the nurse expect to observe?

A. 1
B. 2
C.3
D. 4

A

ANS: A
With decorticate posturing, the patient exhibits internal rotation and adduction of the arms with flexion of the elbows, wrists, and fingers. The other illustrations are of decerebrate, mixed decorticate and decerebrate posturing, and opisthotonic posturing.

How well did you know this?
1
Not at all
2
3
4
5
Perfectly
60
Q

56(39)Which is the correct point on the accompanying figure where the nurse will assess for ecchymosis when admitting a patient with a basilar skull fracture?

a. A
b. B
c. C
d. D

A

ANS: D

Battle’s sign (postauricular ecchymosis) and periorbital ecchymoses are associated with basilar skull fracture.

How well did you know this?
1
Not at all
2
3
4
5
Perfectly
61
Q

An unconscious patient with a traumatic head injury has a blood pressure of 130/76 mm Hg and an intracranial pressure (ICP) of 20 mm Hg. The nurse will calculate the cerebral perfusion pressure (CPP) as ____ mm Hg.

A

ANS: 74
Calculate the CPP: (CPP = Mean arterial pressure [MAP] – ICP). MAP = DBP + 1/3 (Systolic blood pressure [SBP] – Diastolic blood pressure [DBP]). The MAP is 94. The CPP is 74.

How well did you know this?
1
Not at all
2
3
4
5
Perfectly
62
Q

After a patient experienced a brief episode of tinnitus, diplopia, and dysarthria with no residual effects, the nurse anticipates teaching the patient about

A. cerebral aneurysm clipping. c. oral low-dose aspirin therapy.
B. heparin intravenous infusion.
C. Oral low-dose aspirin
D. tissue plasminogen activator (tPA).

A

ANS: C
The patient’s symptoms are consistent with transient ischemic attack (TIA), and drugs that inhibit platelet aggregation are prescribed after a TIA to prevent a stroke. Continuous heparin infusion is not routinely used after TIA or with acute ischemic stroke. The patient’s symptoms are not consistent with a cerebral aneurysm. tPA is used only for acute ischemic stroke, not for TIA

How well did you know this?
1
Not at all
2
3
4
5
Perfectly
63
Q

A patient is being admitted with a possible stroke. Which information from the assessment indicates that the nurse should consult with the health care provider before giving the prescribed aspirin?

a. The patient has dysphasia.
b. The patient has atrial fibrillation.
c. The patient reports that symptoms began with a severe headache.
d. The patient has a history of brief episodes of right-sided hemiplegia

A

ANS: C
A sudden onset headache is typical of a subarachnoid hemorrhage, and aspirin is contraindicated. Atrial fibrillation, dysphasia, and transient ischemic attack are not contraindications to aspirin use

How well did you know this?
1
Not at all
2
3
4
5
Perfectly
64
Q

A patient with a stroke experiences facial drooping on the right side and right-sided arm and leg paralysis. When admitting the patient, which clinical manifestation will the nurse expect to find?

a. Impulsive behavior
b. Right-sided neglect
c. Hyperactive left-sided tendon reflexes
d. Difficulty comprehending instructions

A

ANS: D
Right-sided paralysis indicates a left-brain stroke, which will lead to difficulty with comprehension and use of language. The left-side reflexes are likely to be intact. Impulsive behavior and neglect are more likely with a right-side stroke

65
Q
During the change of shift report, a nurse is told that a patient has an occluded left posterior cerebral artery. The nurse will anticipate that the patient may have
A. dysphasia.
B. confusion. 
C. Visual deficits
D. poor judgment
A

ANS: C
Visual disturbances are expected with posterior cerebral artery occlusion. Aphasia occurs with middle cerebral artery involvement. Cognitive deficits and changes in judgment are more typical of anterior cerebral artery occlusion

66
Q

When teaching about clopidogrel (Plavix), the nurse will tell the patient with cerebral atherosclerosis

a. to monitor and record the blood pressure daily.
b. to call the health care provider if stools are tarry.
c. that clopidogrel will dissolve clots in the cerebral arteries.
d. that clopidogrel will reduce cerebral artery plaque formation

A

ANS: B
Clopidogrel inhibits platelet function and increases the risk for gastrointestinal bleeding, so patients should be advised to notify the health care provider about any signs of bleeding. The medication does not lower blood pressure, decrease plaque formation, or dissolve clots

67
Q

A patient with carotid atherosclerosis asks the nurse to describe a carotid endarterectomy. Which response by the nurse is accurate?

a. “The obstructing plaque is surgically removed from inside an artery in the neck.”
b. “The diseased portion of the artery in the brain is replaced with a synthetic graft.”
c. “A wire is threaded through an artery in the leg to the clots in the carotid artery,
and the clots are removed.”
d. “A catheter with a deflated balloon is positioned at the narrow area, and the
balloon is inflated to flatten the plaque.

A

ANS: A
In a carotid endarterectomy, the carotid artery is incised, and the plaque is removed. The response beginning, “The diseased portion of the artery in the brain is replaced” describes an arterial graft procedure. The answer beginning, “A catheter with a deflated balloon is positioned at the narrow area” describes an angioplasty. The final response beginning, “A wire is threaded through the artery” describes the mechanical embolus removal in cerebral ischemia (MERCI) procedure

68
Q

A patient admitted with possible stroke has been aphasic for 3 hours, and his current blood pressure (BP) is 174/94 mm Hg. Which order by the health care provider should the nurse question?

a. Keep head of bed elevated at least 30 degrees.
b. Infuse normal saline intravenously at 75 mL/hr.
c. Start a labetalol drip to keep BP less than 140/90 mm Hg.
d. Administer tissue plasminogen activator (tPA) intravenously per protocol

A

ANS: C
Because elevated BP may be a protective response to maintain cerebral perfusion, antihypertensive therapy is recommended only if mean arterial pressure (MAP) is greater than130 mm Hg or systolic pressure is greater than 220 mm Hg. Fluid intake should be 1500 to 2000 mL/day to maintain cerebral blood flow. The head of the bed should be elevated to at least 30 degrees unless the patient has symptoms of poor tissue perfusion. tPA may be administered if the patient meets the other criteria for tPA use

69
Q

A patient arrives in the emergency department with hemiparesis and dysarthria that started 2 hours previously, and health records show a history of several transient ischemic attacks (TIAs). The nurse anticipates preparing the patient for

a. surgical endarterectomy.
b. transluminal angioplasty.
c. intravenous heparin drip administration.
d. tissue plasminogen activator (tPA) infusion

A

ANS: D
The patient’s history and clinical manifestations suggest an acute ischemic stroke, and a patient who is seen within 4.5 hours of stroke onset is likely to receive tPA (after screening with a CT scan). Heparin administration in the emergency phase is not indicated. Emergent carotid transluminal angioplasty or endarterectomy is not indicated for the patient who is having an acute ischemiC stroke

70
Q

A female patient who had a stroke 24 hours ago has expressive aphasia. An appropriate nursing intervention to help the patient communicate is to

a. ask questions that the patient can answer with “yes” or “no.”
b. develop a list of words that the patient can read and practice reciting.
c. have the patient practice her facial and tongue exercises with a mirror.
d. prevent embarrassing the patient by answering for her if she does not respond

A

ANS: A
Communication will be facilitated and less frustrating to the patient when questions that require a “yes” or “no” response are used. When the language areas of the brain are injured, the patient might not be able to read or recite words, which will frustrate the patient without improving communication. Expressive aphasia is caused by damage to the language areas of the brain, not by the areas that control the motor aspects of speech. The nurse should allow time for the patient to respond.

71
Q

For a patient who had a right hemisphere stroke, the nurse anticipates planning interventions to manage

a. impaired physical mobility related to right-sided hemiplegia.
b. risk for injury related to denial of deficits and impulsiveness.
c. impaired verbal communication related to speech-language deficits.
d. ineffective coping related to depression and distress about disability.

A

ANS: B
The patient with right-sided brain damage typically denies any deficits and has poor impulse control, leading to risk for injury when the patient attempts activities such as transferring from a bed to a chair. Right-sided brain damage causes left hemiplegia. Left-sided brain damage typically causes language deficits. Left-sided brain damage is associated with depression and distress about the disability

72
Q

When caring for a patient with a new right-sided homonymous hemianopsia resulting from a stroke, which intervention should the nurse include in the plan of care?

a. Apply an eye patch to the right eye.
b. Approach the patient from the right side.
c. Place needed objects on the patient’s left side.
d. Teach the patient that the left visual deficit will resolve

A

ANS: C
During the acute period, the nurse should place objects on the patient’s unaffected side. Because there is a visual defect in the right half of each eye, an eye patch is not appropriate. The patient should be approached from the left side. The visual deficit may not resolve, although the patient can learn to compensate for the defect

73
Q

A left-handed patient with left-sided hemiplegia has difficulty feeding himself. Which intervention should the nurse include in the plan of care?

a. Provide a wide variety of food choices.
b. Provide oral care before and after meals.
c. Assist the patient to eat with the right hand.
d. Teach the patient the “chin-tuck” technique

A

ANS: C
Because the patient has difficulty feeding himself, the appropriate interventions will focus on teaching the patient to use the right hand for self-feeding. The other interventions are appropriate for patients with other etiologies for the imbalanced nutrition

74
Q

A patient has a ruptured cerebral aneurysm and subarachnoid hemorrhage. Which intervention will the nurse include in the plan of care?

a. Apply intermittent pneumatic compression stockings.
b. Assist to dangle on edge of bed and assess for dizziness.
c. Encourage patient to cough and deep breathe every 4 hours.
d. Insert an oropharyngeal airway to prevent airway obstruction

A

ANS: A
The patient with a subarachnoid hemorrhage usually has minimal activity to prevent cerebral vasospasm or further bleeding and is at risk for venous thromboembolism. Activities such as coughing and sitting up that might increase intracranial pressure or decrease cerebral blood flow are avoided. Because there is no indication that the patient is unconscious, an oropharyngeal airway is inappropriate

75
Q

A patient will attempt oral feedings for the first time after having a stroke. The nurse should assess the gag reflex and then

A. order a varied pureed diet.
B. assess the patient’s appetite.
C. Assist the patient into a chair
D. offer the patient a sip of juice

A

ANS: C
The patient should be as upright as possible before attempting feeding to make swallowing easier and decrease aspiration risk. To assess swallowing ability, the nurse should initially offer water or ice to the patient. Pureed diets are not recommended because the texture is too smooth. The patient may have a poor appetite, but the oral feeding should be attempted

76
Q

A male patient who has right-sided weakness after a stroke is making progress in learning to use the left hand for feeding and other activities. The nurse observes that when the patient’s wife is visiting, she feeds and dresses him. Which nursing diagnosis is most appropriate for the patient?

a. Interrupted family processes related to effects of illness of a family member
b. Situational low self-esteem related to increasing dependence on spouse for care
c. Disabled family coping related to inadequate understanding by patient’s spouse
d. Impaired nutrition: less than body requirements related to hemiplegia and aphasia

A

ANS: C
The information supports the diagnosis of disabled family coping because the wife does not understand the rehabilitation program. There are no data supporting low self-esteem, and the patient is attempting independence. The data do not support an interruption in family processes because this may be a typical pattern for the couple. There is no indication that the patient has impaired nutrition

77
Q

Several weeks after a stroke, a 50-yr-old male patient has impaired awareness of bladder fullness, resulting in urinary incontinence. Which nursing intervention should be planned to begin an effective bladder training program?

a. Limit fluid intake to 1200 mL daily to reduce urine volume.
b. Assist the patient onto the bedside commode every 2 hours.
c. Perform intermittent catheterization after each voiding to check for residual urine.
d. Use an external “condom” catheter to protect the skin and prevent embarrassment

A

ANS: B
Developing a regular voiding schedule will prevent incontinence and may increase patient awareness of a full bladder. A 1200-mL fluid restriction may lead to dehydration. Intermittent catheterization and use of a condom catheter are appropriate in the acute phase of stroke, but should not be considered solutions for long-term management because of the risks for urinary tract infection and skin breakdown.

78
Q

. A patient who has a history of a transient ischemic attack (TIA) has an order for aspirin 160 mg daily. When the nurse is administering medications, the patient says, “I don’t need the aspirin today. I don’t have a fever.” Which action should the nurse take?

a. Document that the aspirin was refused by the patient.
b. Tell the patient that the aspirin is used to prevent a fever.
c. Explain that the aspirin is ordered to decrease stroke risk.
d. Call the health care provider to clarify the medication order

A

ANS: C
Aspirin is ordered to prevent stroke in patients who have experienced TIAs. Documentation of the patient’s refusal to take the medication is an inadequate response by the nurse. There is no need to clarify the order with the health care provider. The aspirin is not ordered to prevent aches and pains

79
Q

A patient in the clinic reports a recent episode of dysphasia and left-sided weakness at home that resolved after 2 hours. The nurse will anticipate teaching the patient about

A. tPA.
B. aspirin .
C. Warfarin (Coumadin)
D. nimodipine

A

ANS: B
After a transient ischemic attack, patients typically are started on medications such as aspirin to inhibit platelet function and decrease stroke risk. tPA is used for acute ischemic stroke. Coumadin is usually used for patients with atrial fibrillation. Nimodipine is used to prevent cerebral vasospasm after a subarachnoid hemorrhage

80
Q

A patient with a left-brain stroke suddenly bursts into tears when family members visit. The nurse should

a. use a calm voice to ask the patient to stop the crying behavior.
b. explain to the family that depression is normal following a stroke.
c. have the family members leave the patient alone for a few minutes.
d. teach the family that emotional outbursts are common after strokes

A

ANS: D
Patients who have left-sided brain stroke are prone to emotional outbursts that are not necessarily related to the emotional state of the patient. Depression after a stroke is common, but the suddenness of the patient’s outburst suggests that depression is not the major cause of the behavior. The family should stay with the patient. The crying is not within the patient’s control, and asking the patient to stop will lead to embarrassment

81
Q

Which stroke risk factor for a 48-yr-old male patient in the clinic is most important for the nurse to address?

a. The patient is 25 lb above the ideal weight.
b. The patient drinks a glass of red wine with dinner daily.
c. The patient’s usual blood pressure (BP) is 170/94 mm Hg.
d. The patient works at a desk and relaxes by watching television

A

ANS: C
Hypertension is the single most important modifiable risk factor. People who drink more than 1 (for women) or 2 (for men) alcoholic beverages a day may increase their risk for hypertension. Physical inactivity and obesity contribute to stroke risk but not as much as hypertension

82
Q

A patient in the emergency department with sudden-onset right-sided weakness is diagnosed with an intracerebral hemorrhage. Which information about the patient is most important to communicate to the health care provider?

a. The patient’s speech is difficult to understand.
b. The patient’s blood pressure (BP) is 144/90 mm Hg.
c. The patient takes a diuretic because of a history of hypertension.
d. The patient has atrial fibrillation and takes warfarin (Coumadin)

A

ANS: D
The use of warfarin probably contributed to the intracerebral bleeding and remains a risk factor for further bleeding. Administration of vitamin K is needed to reverse the effects of the warfarin, especially if the patient is to have surgery to correct the bleeding. The history of hypertension is a risk factor for the patient but has no immediate effect on the patient’s care. The BP of 144/90 indicates the need for ongoing monitoring but not for any immediate change in therapy. Slurred speech is consistent with a left-sided stroke, and no change in therapy is indicated

83
Q

A patient with left-sided weakness that started 60 minutes earlier is admitted to the emergency department and diagnostic tests are ordered. Which test should be done first?

a. Complete blood count (CBC)
b. Chest radiograph (chest x-ray)
c. Computed tomography (CT) scan
d. 12-Lead electrocardiogram (EEG)

A

ANS: C
Rapid screening with a noncontrast CT scan is needed before administration of tissue plasminogen activator (tPA), which must be given within 4.5 hours of the onset of clinical manifestations of the stroke. The sooner the tPA is given, the less brain injury. The other diagnostic tests give information about possible causes of the stroke and do not need to be completed as urgently as the CT scan

84
Q

Nurses in change-of-shift report are discussing the care of a patient with a stroke who has progressively increasing weakness and decreasing level of consciousness. Which patient problem do they determine has the highest priority for the patient?

a. Impaired physical mobility related to weakness
b. Disturbed sensory perception related to brain injury
c. Risk for impaired skin integrity related to immobility
d. Risk for aspiration related to inability to protect airway

A

ANS: D
Protection of the airway is the priority of nursing care for a patient having an acute stroke. The other diagnoses are also appropriate, but interventions to prevent aspiration are the priority at this time

85
Q

Which information about the patient who has had a subarachnoid hemorrhage is most important to communicate to the health care provider?

a. The patient complains of having a stiff neck.
b. The patient’s blood pressure (BP) is 90/50 mm Hg.
c. The patient reports a severe and unrelenting headache.
d. The cerebrospinal fluid (CSF) report shows red blood cells (RBCs).

A

ANS: B
To prevent cerebral vasospasm and maintain cerebral perfusion, BP needs to be maintained at a level higher than 90 mm Hg systolic after a subarachnoid hemorrhage. A low BP or drop in BP indicates a need to administer fluids and/or vasopressors to increase the BP. An ongoing headache, RBCs in the CSF, and a stiff neck are all typical clinical manifestations of a subarachnoid hemorrhage and do not need to be rapidly communicated to the health care provider.

86
Q

The nurse is caring for a patient who has been experiencing stroke symptoms for 60 minutes. Which action can the nurse delegate to a licensed practical/vocational nurse (LPN/LVN)?

a. Assess the patient’s gag and cough reflexes.
b. Determine when the stroke symptoms began.
c. Administer the prescribed short-acting insulin. d. Infuse the prescribed IV metoprolol (Lopressor)

A

ANS: C
Administration of subcutaneous medications is included in LPN/LVN education and scope of practice. The other actions require more education and scope of practice and should be done by the registered nurse (RN)

87
Q

After receiving change-of-shift report on the following four patients, which patient should the nurse see first?

a. A 60-yr-old patient with right-sided weakness who has an infusion of tPA
prescribed
b. A 50-yr-old patient who has atrial fibrillation and a new order for warfarin
(Coumadin)
c. A 30-yr-old patient with a subarachnoid hemorrhage 2 days ago who has
nimodipine scheduled
d. A 40-yr-old patient who experienced a transient ischemic attack yesterday who
has a dose of aspirin due.

A

ANS: A
tPA needs to be infused within the first few hours after stroke symptoms start in order to be effective in minimizing brain injury. The other medications should also be given as quickly as possible, but timing of the medications is not as critical

88
Q

The nurse is caring for a patient who has just returned after having left carotid artery angioplasty and stenting. Which assessment information is of most concern to the nurse?

a. The pulse rate is 102 beats/min.
b. The patient has difficulty speaking.
c. The blood pressure is 144/86 mm Hg.
d. There are fine crackles at the lung bases

A

ANS: B
Small emboli can occur during carotid artery angioplasty and stenting, and the aphasia indicates a possible stroke during the procedure. Slightly elevated pulse rate and blood pressure are not unusual because of anxiety associated with the procedure. Fine crackles at the lung bases may indicate atelectasis caused by immobility during the procedure. The nurse should have the patient take some deep breaths

89
Q

A 70-yr-old female patient with left-sided hemiparesis arrives by ambulance to the emergency department. Which action should the nurse take first?

a. Take the patient’s blood pressure.
b. Check the respiratory rate and effort.
c. Assess the Glasgow Coma Scale score.
d. Send the patient for a computed tomography (CT) scan

A

ANS: B
The initial nursing action should be to assess the airway and take any needed actions to ensure a patent airway. The other activities should take place quickly after the ABCs (airway, breathing, and circulation) are completed.

90
Q

A 63-yr-old patient who began experiencing right arm and leg weakness is admitted to the emergency department. In which order will the nurse implement these actions included in the stroke protocol? (Put a comma and a space between each answer choice [A, B, C, D].)

a. Obtain CT scan without contrast.
b. Infuse tissue plasminogen activator (tPA).
c. Administer oxygen to keep O2 saturation >95%.
d. Use National Institute of Health Stroke Scale to assess patient

A

ANS:
C, D, A, B
The initial actions should be those that help with airway, breathing, and circulation. Baseline neurologic assessments should be done next. A CT scan will be needed to rule out hemorrhagic stroke before tPA can be administered

91
Q

The nurse determines that teaching about management of migraine headaches has been effective when the patient says which of the following?

a. “I can take the (Topamax) as soon as a headache starts.”
b. “A glass of wine might help me relax and prevent a headache.”
c. “I will lie down someplace dark and quiet when the headaches begin.”
d. “I should avoid taking aspirin and sumatriptan (Imitrex) at the same time.”

A

ANS: C
It is recommended that the patient with a migraine rest in a dark, quiet area. Topiramate (Topamax) is used to prevent migraines and must be taken for several months to determine effectiveness. Aspirin or other nonsteroidal antiinflammatory medications can be taken with the triptans. Alcohol may precipitate migraine headaches.

92
Q
The nurse expects the assessment of a patient who is experiencing a cluster headache to include
A. nuchal rigidity. 
B.unilateral ptosis. 
C. Projectile vomiting
D. throbbing, bilateral facial pain.
A

ANS: B
Unilateral eye edema, tearing, and ptosis are characteristic of cluster headaches. Nuchal rigidity suggests meningeal irritation, such as occurs with meningitis. Although nausea and vomiting may occur with migraine headaches, projectile vomiting is more consistent with increased intracranial pressure. Unilateral sharp, stabbing pain, rather than throbbing pain, is charact eristic of cluster headaches.

93
Q

While the nurse is transporting a patient on a stretcher to the radiology department, the patient begins having a tonic-clonic seizure. Which action should the nurse take?

a. Insert an oral airway during the seizure to maintain a patent airway.
b. Restrain the patient’s arms and legs to prevent injury during the seizure.
c. Time and observe and record the details of the seizure and postictal state.
d. Avoid touching the patient to prevent further nervous system stimulation.

A

ANS: C
Because the diagnosis and treatment of seizures frequently are based on the description of the seizure, recording the length and details of the seizure is important. Insertion of an oral airway and restraining the patient during the seizure are contraindicated. The nurse may need to move the patient to decrease the risk of injury during the seizure.

94
Q

A high school teacher who has been diagnosed with epilepsy after having a generalized tonic-clonic seizure tells the nurse, “I cannot teach any more. It will be too upsetting if I have a seizure at work.” Which response by the nurse specifically addresses the patient’s concern?

a. “You might benefit from some psychologic counseling.”
b. “Epilepsy usually can be well controlled with medications.”
c. “You will want to contact the Epilepsy Foundation for assistance.”
d. “The Department of Vocational Rehabilitation can help with work retraining.”

A

ANS: B
The nurse should inform the patient that most patients with seizure disorders are controlled with medication. The other information may be necessary if the seizures persist after treatment with antiseizure medications is implemented

95
Q
A patient has been taking phenytoin (Dilantin) for 2 years. Which action will the nurse take when evaluating for adverse effects of the medication?
A. Inspect the oral mucosa. 
B. Listen to the lung sounds.
C. Auscultaste the bowel sounds.
D. Check pupil reaction to light.
A

ANS: A

Phenytoin can cause gingival hyperplasia, but does not affect bowel sounds, lung sounds, or pupil reaction to light.

96
Q
A patient reports feeling numbness and tingling of the left arm before experiencing a seizure. The nurse determines that this history is consistent with what type of seizure?
A. Focal
B. Atonic 
C. Absence
D. Myoclonic
A

ANS: A
The initial symptoms of a focal seizure involve clinical manifestations that are localized to a particular part of the body or brain. Symptoms of an absence seizure are staring and a brief loss of consciousness. In an atonic seizure, the patient loses muscle tone and (typically) falls to the ground. Myoclonic seizures are characterized by a sudden jerk of the body or extremities.

97
Q

When obtaining a health history and physical assessment for a 36-yr-old female patient with possible multiple sclerosis (MS), the nurse should

a. assess for the presence of chest pain.
b. inquire about urinary tract problems.
c. inspect the skin for rashes or discoloration.
d. ask the patient about any increase in libido

A

ANS: B
Urinary tract problems with incontinence or retention are common symptoms of MS. Chest pain and skin rashes are not symptoms of MS. A decrease in libido is common with MS

98
Q

A woman who has multiple sclerosis (MS) asks the nurse about risks associated with pregnancy. Which response by the nurse is accurate?

a. “MS symptoms may be worse after the pregnancy.”
b. “Women with MS frequently have premature labor.”
c. “MS is associated with an increased risk for congenital defects.”
d. “Symptoms of MS are likely to become worse during pregnancy.”

A

ANS: A
During the postpartum period, women with MS are at greater risk for exacerbation of symptoms. There is no increased risk for congenital defects in infants born of mothers with MS. Symptoms of MS may improve during pregnancy. Onset of labor is not affected by MS.

99
Q

A 33-yr-old patient with multiple sclerosis (MS) is to begin treatment with glatiramer acetate (Copaxone). Which information will the nurse include in patient teaching?

a. Recommendation to drink at least 4 L of fluid daily
b. Need to avoid driving or operating heavy machinery
c. How to draw up and administer injections of the medication
d. Use of contraceptive methods other than oral contraceptives

A

ANS: C
Copaxone is administered by self-injection. Oral contraceptives are an appropriate choice for birth control. There is no need to avoid driving or drink large fluid volumes when taking glatiramer

100
Q

Which information about a 60-yr-old patient with multiple sclerosis indicates that the nurse should consult with the health care provider before giving the prescribed dose of dalfampridine (Ampyra)?

a. The patient walks a mile each day for exercise.
b. The patient complains of pain with neck flexion.
c. The patient has an increased serum creatinine level.
d. The patient has the relapsing-remitting form of MS.

A

ANS: C
Dalfampridine should not be given to patients with impaired renal function. The other information will not impact whether the dalfampridine should be administered.

101
Q

Which action will the nurse plan to take for a patient with multiple sclerosis who has urinary retention caused by a flaccid bladder?

a. Encourage a decreased evening intake of fluid.
b. Teach the patient how to use the Credé method.
c. Suggest the use of adult incontinence briefs for nighttime only.
d. Assist the patient to the commode every 2 hours during the day.

A

ANS: B
The Credé method can be used to improve bladder emptying. Decreasing fluid intake will not improve bladder emptying and may increase risk for urinary tract infection and dehydration. The use of incontinence briefs and frequent toileting will not improve bladder emptying.

102
Q

A patient with Parkinson’s disease has bradykinesia. Which action will the nurse include in the plan of care?

a. Instruct the patient in activities that can be done while lying or sitting.
b. Suggest that the patient rock from side to side to initiate leg movement.
c. Have the patient take small steps in a straight line directly in front of the feet.
d. Teach the patient to keep the feet in contact with the floor and slide them forward.

A

ANS: B
Rocking the body from side to side stimulates balance and improves mobility. The patient will be encouraged to continue exercising because this will maintain functional abilities. Maintaining a wide base of support will help with balance. The patient should lift the feet and avoid a shuffling gait.

103
Q

A 62-yr-old patient who has Parkinson’s disease is taking bromocriptine (Parlodel). Which information obtained by the nurse may indicate a need for a decrease in the dosage?

a. The patient has a chronic dry cough.
b. The patient has four loose stools in a day.
c. The patient develops a deep vein thrombosis.
d. The patient’s blood pressure is 92/52 mm Hg.

A

ANS: D
Hypotension is an adverse effect of bromocriptine, and the nurse should check with the health care provider before giving the medication. Diarrhea, cough, and deep vein thrombosis are not associated with bromocriptine use

104
Q

The nurse advises a patient with myasthenia gravis (MG) to

a. perform physically demanding activities early in the day.
b. anticipate the need for weekly plasmapheresis treatments.
c. do frequent weight-bearing exercise to prevent muscle atrophy.
d. protect the extremities from injury due to poor sensory perception

A

ANS: A
Muscles are generally strongest in the morning, and activities involving muscle activity should be scheduled then. Plasmapheresis is not routinely scheduled but is used for myasthenia crisis or for situations in which corticosteroid therapy must be avoided. There is no decrease in sensation with MG, and muscle atrophy does not occur because although there is muscle weakness, they are still used.

105
Q
Which medication taken by a patient with restless legs syndrome should the nurse discuss with the patient? 
A. Ibuprofen 
B. Multivitamin 
C. Acetaminophen 
D. Diphenhydramine
A

ANS: D
Antihistamines can aggravate restless legs syndrome. The other medications will not contribute to restless legs syndrome.

106
Q

A patient who has amyotrophic lateral sclerosis (ALS) is hospitalized with pneumonia. Which nursing action will be included in the plan of care?

a. Observe for agitation and paranoia.
b. Assist with active range of motion (ROM).
c. Give muscle relaxants as needed to reduce spasms.
d. Use simple words and phrases to explain procedures

A

ANS: B
ALS causes progressive muscle weakness, but assisting the patient to perform active ROM will help maintain strength as long as possible. Psychotic manifestations such as agitation and paranoia are not associated with ALS. Cognitive function is not affected by ALS, and the patient’s ability to understand procedures will not be impaired. Muscle relaxants will further increase muscle weakness and depress respirations.

107
Q

A 40-yr-old patient is diagnosed with early Huntington’s disease (HD). When teaching the patient, spouse, and adult children about this disorder, the nurse will provide information about the

a. use of levodopa-carbidopa (Sinemet) to help reduce HD symptoms.
b. prophylactic antibiotics to decrease the risk for aspiration pneumonia.
c. option of genetic testing for the patient’s children to determine their own HD risks.
d. lifestyle changes of improved nutrition and exercise that delay disease progression.

A

ANS: C
Genetic testing is available to determine whether an asymptomatic individual has the HD gene. The patient and family should be informed of the benefits and problems associated with genetic testing. Sinemet will increase symptoms of HD because HD involves an increase in dopamine. Antibiotic therapy will not reduce the risk for aspiration. There are no effective treatments or lifestyle changes that delay the progression of symptoms in HD.

108
Q

When a 74-yr-old patient is seen in the health clinic with new development of a stooped posture, shuffling gait, and pill rolling–type tremor, the nurse will anticipate teaching the patient about

a. oral corticosteroids.
b. antiparkinsonian drugs.
c. magnetic resonance imaging (MRI).
d. electroencephalogram (EEG) testing

A

ANS: B
The clinical diagnosis of Parkinson’s is made when tremor, rigidity, and akinesia, and postural instability are present. The confirmation of the diagnosis is made on the basis of improvement when antiparkinsonian drugs are administered. MRI and EEG are not useful in diagnosing Parkinson’s disease, and corticosteroid therapy is not used to treat it.

109
Q

A 22-yr-old patient seen at the health clinic with a severe migraine headache tells the nurse about having similar headaches recently. Which initial action should the nurse take?

a. Teach about the use of triptan drugs.
b. Refer the patient for stress counseling.
c. Ask the patient to keep a headache diary.
d. Suggest the use of muscle-relaxation techniques.

A

ANS: C
The initial nursing action should be further assessment of the precipitating causes of the headaches, quality, and location of pain. Stress reduction, muscle relaxation, and the triptan drugs may be helpful, but more assessment is needed first

110
Q
A hospitalized patient complains of a bilateral headache (4/10 on the pain scale) that radiates from the base of the skull. Which prescribed PRN medications should the nurse administer initially?
A. Lorazepam (Ativan) 
B. Acetaminophen (Tylenol) 
C. Morphine sulfate (MS Continue)
D. Butalbital and aspirin (Fiorinal)
A

ANS: B
The patient’s symptoms are consistent with a tension headache, and initial therapy usually involves a nonopioid analgesic such as acetaminophen, which is sometimes combined with a sedative or muscle relaxant. Lorazepam may be used in conjunction with acetaminophen but would not be appropriate as the initial monotherapy. Morphine sulfate and butalbital and aspirin would be more appropriate for a headache that did not respond to a nonopioid analgesic

111
Q

A patient tells the nurse about using acetaminophen (Tylenol) several times every day for recurrent bilateral headaches. Which action will the nurse plan to take first?

a. Discuss the need to stop taking the acetaminophen.
b. Suggest the use of biofeedback for headache control.
c. Describe the use of botulism toxin (Botox) for headaches.
d. Teach the patient about magnetic resonance imaging (MRI).

A

ANS: A
The headache description suggests that the patient is experiencing medication overuse headache. The initial action will be withdrawal of the medication. The other actions may be needed if the headaches persist.

112
Q

The health care provider is considering the use of sumatriptan (Imitrex) for a 54-yr-old male patient with migraine headaches. Which information obtained by the nurse is most important to report to the health care provider?

a. The patient drinks 1 to 2 cups of coffee daily.
b. The patient had a recent acute myocardial infarction.
c. The patient has had migraine headaches for 30 years.
d. The patient has taken topiramate (Topamax) for 2 months.

A

ANS: B
The triptans cause coronary artery vasoconstriction and should be avoided in patients with coronary artery disease. The other information will be reported to the health care provider, but none of it indicates that sumatriptan would be an inappropriate treatment.

113
Q

The nurse observes a patient ambulating in the hospital hall when the patient’s arms and legs suddenly jerk and the patient falls to the floor. The nurse will first

a. assess the patient for a possible injury.
b. give the scheduled divalproex (Depakote).
c. document the timing and description of the seizure.
d. notify the patient’s health care provider about the seizure.

A

ANS: A
The patient who has had a myoclonic seizure and fall is at risk for head injury and should first be evaluated and treated for this possible complication. Documentation of the seizure, notification of the health care provider, and administration of antiseizure medications are also appropriate actions, but the initial action should be assessment for injury.

114
Q

Which prescribed intervention will the nurse implement first for a patient in the emergency department who is experiencing continuous tonic-clonic seizures?

a. Give phenytoin (Dilantin) 100 mg IV.
b. Monitor level of consciousness (LOC).
c. Administer lorazepam (Ativan) 4 mg IV.
d. Obtain computed tomography (CT) scan.

A

ANS: C
To prevent ongoing seizures, the nurse should administer rapidly acting antiseizure medications such as the benzodiazepines. A CT scan is appropriate, but prevention of any seizure activity during the CT scan is necessary. Phenytoin will also be administered, but it is not rapidly acting. Patients who are experiencing tonic-clonic seizures are nonresponsive, although the nurse should assess LOC after the seizure.

115
Q

The home health registered nurse (RN) is planning care for a patient with a seizure disorder related to a recent head injury. Which nursing action can be delegated to a licensed practical/vocational nurse (LPN/LVN)?

a. Make referrals to appropriate community agencies.
b. Place medications in the home medication organizer.
c. Teach the patient and family how to manage seizures.
d. Assess for use of medications that may precipitate seizures

A

ANS: B
LPN/LVN education includes administration of medications. The other activities require RN education and scope of practice.

116
Q
A patient is being treated with carbidopa/levodopa (Sinemet) for Parkinson’s disease. Which information indicates a need for change in the medication or dosage?
A. Shuffling gait 
B. Tremor at rest 
C. Cogwheel rigidity of limbs
D. Uncontrolled head movement
A

ANS: D
Dyskinesia is an adverse effect of the Sinemet, indicating a need for a change in medication or decrease in dose. The other findings are typical with Parkinson’s disease.

117
Q

Which nursing diagnosis is of highest priority for a patient with Parkinson’s disease who is unable to move the facial muscles?

a. Activity intolerance
b. Self-care deficit: toileting
c. Ineffective self-health management
d. Imbalanced nutrition: less than body requirements

A

ANS: D
The data about the patient indicate that poor nutrition will be a concern because of decreased swallowing. The other diagnoses may also be appropriate for a patient with Parkinson’s disease, but the data do not indicate that they are current problems for this patient.

118
Q
Which assessment is most important for the nurse to make regarding a patient with myasthenia gravis?
A. Pupil size 
B. Grip strength 
C. Respiratory effort 
D. Level of consciousness
A

ANS: C
Because respiratory insufficiency may be life threatening, it will be most important to monitor respiratory function. The other data also will be assessed but are not as critical.

119
Q

After a thymectomy, a patient with myasthenia gravis receives the usual dose of pyridostigmine (Mestinon). An hour later, the patient complains of nausea and severe abdominal cramps. Which action should the nurse take first?

a. Auscultate the patient’s bowel sounds.
b. Notify the patient’s health care provider.
c. Administer the prescribed PRN antiemetic drug.
d. Give the scheduled dose of prednisone (Deltasone)

A

ANS: B
The patient’s history and symptoms indicate a possible cholinergic crisis. The health care provider should be notified immediately, and it is likely that atropine will be prescribed. The other actions will be appropriate if the patient is not experiencing a cholinergic crisis.

120
Q

A hospitalized patient with a history of cluster headache awakens during the night with a severe stabbing headache. Which action should the nurse take first?

a. Put a moist hot pack on the patient’s neck.
b. Start the prescribed PRN O2 at 6 L/min.
c. Give the ordered PRN acetaminophen (Tylenol).
d. Notify the patient’s health care provider immedIately

A

ANS: B
Acute treatment for cluster headache is administration of 100% O2 at 6 to 8 L/min. If the patient obtains relief with the O2, there is no immediate need to notify the health care provider. Cluster headaches last only 60 to 90 minutes, so oral pain medications have minimal effect. Hot packs are helpful for tension headaches but are not as likely to reduce pain associated with a cluster headache

121
Q

. Which intervention will the nurse include in the plan of care for a patient with primary restless legs syndrome (RLS) who is having difficulty sleeping?

a. Teach about the use of antihistamines to improve sleep.
b. Suggest that the patient exercise regularly during the day.
c. Make a referral to a massage therapist for deep massage of the legs.
d. Assure the patient that the problem is transient and likely to resolve.

A

ANS: B
Nondrug interventions such as getting regular exercise are initially suggested to improve sleep quality in patients with RLS. Antihistamines may aggravate RLS. Massage does not alleviate RLS symptoms, and RLS is likely to progress in most patients.

122
Q

Which information about a patient who has a new prescription for phenytoin (Dilantin) indicates that the nurse should consult with the health care provider before administration of the medication?

a. Patient has tonic-clonic seizures.
b. Patient experiences an aura before seizures.
c. Patient has minor elevations in the liver function tests.
d. Patient’s most recent blood pressure is 156/92 mm Hg.

A

ANS: C
Many older patients (especially with compromised liver function) may not be able to metabolize phenytoin. The health care provider may need to choose another antiseizure medication. Phenytoin is an appropriate medication for patients with tonic-clonic seizures, with or without an aura. Hypertension is not a contraindication for phenytoin therapy.

123
Q

After change-of-shift report, which patient should the nurse assess first?

a. Patient with myasthenia gravis who is reporting increased muscle weakness
b. Patient with a bilateral headache described as “like a band around my head”
c. Patient with seizures who is scheduled to receive a dose of phenytoin (Dilantin)
d. Patient with Parkinson’s disease who has developed cogwheel rigidity of the arms

A

ANS: A
Because increased muscle weakness may indicate the onset of a myasthenic crisis, the nurse should assess this patient first. The other patients should also be assessed but do not appear to need immediate nursing assessments or actions to prevent life-threatening complications.

124
Q

A patient who has been treated for status epilepticus in the emergency department will be transferred to the medical nursing unit. Which equipment should the nurse have available in the patient’s assigned room (select all that apply)?

a. Side-rail pads
b. Tongue blade
c. Oxygen mask
d. Suction tubing
e. Urinary catheter
f. Nasogastric tube

A

ANS: A, C, D
The patient is at risk for further seizures, and O2 and suctioning may be needed after any seizures to clear the airway and maximize oxygenation. The bed’s side rails should be padded to minimize the risk for patient injury during a seizure. Use of tongue blades during a seizure is contraindicated. Insertion of a nasogastric (NG) tube is not indicated because the airway problem is not caused by vomiting or abdominal distention. A urinary catheter is not required unless there is urinary retention

125
Q

A patient with Parkinson’s disease is admitted to the hospital for treatment of pneumonia. Which nursing interventions will be included in the plan of care (select all that apply)?

a. Provide an elevated toilet seat.
b. Cut patient’s food into small pieces.
c. Serve high-protein foods at each meal.
d. Place an armchair at the patient’s bedside.
e. Observe for sudden exacerbation of symptoms.

A

ANS: A, B, D
Because the patient with Parkinson’s disease has difficulty chewing, food should be cut into small pieces. An armchair should be used when the patient is seated so that the patient can use the arms to assist with getting up from the chair. An elevated toilet seat will facilitate getting on and off the toilet. High-protein foods will decrease the effectiveness of L-dopa. Parkinson’s disease is a steadily progressive disease without acute exacerbations.

126
Q

A patient who is having an acute exacerbation of multiple sclerosis has a prescription for methylprednisolone (Solu-Medrol) 150 mg IV. The label on the vial reads: methylprednisolone 125 mg in 2 mL. How many mL will the nurse administer?

A

ANS: 2.4

With a concentration of 125 mg/2 mL, the nurse will need to administer 2.4 mL to obtain 150 mg of methylprednisolone.

127
Q

The nurse assessing a patient with newly diagnosed trigeminal neuralgia will ask the patient about

a. visual problems caused by ptosis.
b. triggers leading to facial discomfort.
c. poor appetite caused by loss of taste.
d. weakness on the affected side of the face.

A

ANS: B
The major clinical manifestation of trigeminal neuralgia is severe facial pain triggered by cutaneous stimulation of the nerve. Ptosis, loss of taste, and facial weakness are not characteristics of trigeminal neuralgia.

128
Q

Which patient assessment will help the nurse identify potential complications of trigeminal neuralgia?

a. Have the patient clench the jaws.
b. Inspect the oral mucosa and teeth.
c. Palpate the face to compare skin temperature bilaterally.
d. Identify trigger zones by lightly touching the affected side.

A

ANS: B
Oral hygiene is frequently neglected because of fear of triggering facial pain and may lead to gum disease, dental caries, or an abscess. Having the patient clench the facial muscles will not be useful because the sensory branches (rather than motor branches) of the nerve are affected by trigeminal neuralgia. Light touch and palpation may be triggers for pain and should be avoided.

129
Q

When evaluating outcomes of a glycerol rhizotomy for a patient with trigeminal neuralgia, the nurse will

a. assess if the patient is doing daily facial exercises.
b. question if the patient is using an eye shield at night.
c. ask the patient about social activities with family and friends.
d. remind the patient to chew on the unaffected side of the mouth.

A

ANS: C
Because withdrawal from social activities is a common manifestation of trigeminal neuralgia, asking about social activities will help in evaluating if the patient’s symptoms have improved. Glycerol rhizotomy does not damage the corneal reflex or motor functions of the trigeminal nerve, so there is no need to use an eye shield, do facial exercises, or take precautions with chewing.

130
Q

Which action will the nurse include in the plan of care for a patient who is experiencing pain from trigeminal neuralgia?

a. Assess fluid and dietary intake.
b. Apply ice packs for 20 minutes.
c. Teach facial relaxation techniques.
d. Spend time talking with the patient.

A

ANS: A
The patient with an acute episode of trigeminal neuralgia may be unwilling to eat or drink, so assessment of nutritional and hydration status is important. Because stimulation by touch is the precipitating factor for pain, relaxation of the facial muscles will not improve symptoms. Application of ice is likely to precipitate pain. The patient will not want to engage in conversation, which may precipitate attacks.

131
Q

The nurse identifies a patient with type 1 diabetes and a history of herpes simplex infection as being at risk for Bell’s palsy. Which information should the nurse include in teaching the patient?

a. “You may be able to prevent Bell’s palsy by doing facial exercises regularly.”
b. “Prophylactic treatment of herpes with antiviral agents prevents Bell’s palsy.”
c. “Medications to treat Bell’s palsy work only if started before paralysis onset.”
d. “Call the doctor if you experience pain or develop herpes lesions near the ear.”

A

ANS: D
Pain or herpes lesions near the ear may indicate the onset of Bell’s palsy, and rapid corticosteroid treatment may reduce the duration of Bell’s palsy symptoms. Antiviral therapy for herpes simplex does not reduce the risk for Bell’s palsy. Corticosteroid therapy will be most effective in reducing symptoms if started before paralysis is complete but will still be somewhat effective when started later. Facial exercises do not prevent Bell’s palsy.

132
Q

A patient with Bell’s palsy refuses to eat while others are present because of embarrassment about drooling. The best response by the nurse is to

a. respect the patient’s feelings and arrange for privacy at mealtimes.
b. teach the patient to chew food on the unaffected side of the mouth.
c. offer the patient liquid nutritional supplements at frequent intervals.
d. discuss the patient’s concerns with visitors who arrive at mealtimes.

A

ANS: A
The patient’s desire for privacy should be respected to encourage adequate nutrition and reduce patient embarrassment. Liquid supplements may help maintain nutrition but will reduce the patient’s enjoyment of the taste of food. It would be inappropriate for the nurse to discuss the patient’s embarrassment with visitors unless the patient wishes to share this information. Chewing on the unaffected side of the mouth will enhance nutrition and enjoyment of food but will not decrease the drooling.

133
Q

To prevent autonomic hyperreflexia, which nursing action will the home health nurse include in the plan of care for a patient who has paraplegia at the T4 level ?

a. Support selection of a high-protein diet.
b. Discuss options for sexuality and fertility.
c. Assist in planning a prescribed bowel program.
d. Use quad coughing to strengthen cough efforts.

A

ANS: C
Fecal impaction is a common stimulus for autonomic hyperreflexia. Dietary protein, coughing, and discussing sexuality and fertility should be included in the plan of care but will not reduce the risk for autonomic hyperreflexia.

134
Q

Which assessment data for a patient who has Guillain-Barré syndrome will require the nurse’s most immediate action?

a. The patient’s sacral area skin is reddened.
b. The patient is continuously drooling saliva.
c. The patient complains of severe pain in the feet.
d. The patient’s blood pressure (BP) is 150/82 mm Hg.

A

ANS: B
Drooling indicates decreased ability to swallow, which places the patient at risk for aspiration and requires rapid nursing and collaborative actions such as suctioning and possible endotracheal intubation. The foot pain should be treated with appropriate analgesics, the BP requires ongoing monitoring, and the skin integrity requires intervention, but these actions are not as urgently needed as maintenance of respiratory function.

135
Q

. A patient hospitalized with a new diagnosis of Guillain-Barré syndrome has numbness and weakness of both feet. The nurse will anticipate teaching the patient about

a. infusion of immunoglobulin
b. intubation and mechanical ventilation.
c. administration of corticosteroid drugs.
d. insertion of a nasogastric (NG) feeding tube.

A

ANS: A
Because Guillain-Barré syndrome is in the earliest stages (as evidenced by the symptoms), use of high-dose immunoglobulin is appropriate to reduce the extent and length of symptoms. Mechanical ventilation and tube feedings may be used later in the progression of the syndrome but are not needed now. Corticosteroid use is not helpful in reducing the duration or symptoms of the syndrome.

136
Q

A construction worker arrives at an urgent care center with a deep puncture wound from a rusty nail. The patient reports having had a tetanus booster 6 years ago. The nurse will anticipate

a. IV infusion of tetanus immune globulin (TIG).
b. administration of the tetanus-diphtheria (Td) booster.
c. intradermal injection of an immune globulin test dose.
d. initiation of the tetanus-diphtheria immunization series.

A

ANS: B
If the patient has not been immunized in the past 5 years, administration of the Td booster is indicated because the wound is deep. Immune globulin administration is given by the IM route if the patient has no previous immunization. Administration of a series of immunization is not indicated. TIG is not indicated for this patient, and a test dose is not needed for immune globulin.

137
Q

The nurse is admitting a patient who has a neck fracture at the C6 level to the intensive care unit. Which assessment findings indicate neurogenic shock?

a. Involuntary and spastic movement
b. Hypotension and warm extremities
c. Hyperactive reflexes below the injury
d. Lack of sensation or movement below the injury

A

ANS: B
Neurogenic shock is characterized by hypotension, bradycardia, and vasodilation leading to warm skin temperature. Spasticity and hyperactive reflexes do not occur at this stage of spinal cord injury. Lack of movement and sensation indicate spinal cord injury but not neurogenic shock.

138
Q

A patient has an incomplete left spinal cord lesion at the level of T7, resulting in Brown-Séquard syndrome. Which nursing action should be included in the plan of care?

a. Assessment of the patient for right arm weakness
b. Assessment of the patient for increased right leg pain
c. Positioning the patient’s left leg when turning the patient
d. Teaching the patient to look at the right leg to verify its position

A

ANS: C
The patient with Brown-Séquard syndrome has loss of motor function on the ipsilateral side and will require the nurse to move the left leg. Pain sensation will be lost in the patient’s right leg. Arm weakness will not be a problem for a patient with a T7 injury. The patient will retain position sense for the right leg.

139
Q

The nurse will explain to the patient who has a T2 spinal cord transection injury that

a. use of the shoulders will be limited.
b. function of both arms should be retained.
c. total loss of respiratory function may occur.
d. tachycardia is common with this type of injury.

A

ANS: B
The patient with a T2 injury can expect to retain full motor and sensory function of the arms. Use of only the shoulders is associated with cervical spine injury. Loss of respiratory function occurs with cervical spine injuries. Bradycardia is associated with injuries above the T6 level.

140
Q

A patient with paraplegia resulting from a T9 spinal cord injury has a neurogenic reflexic bladder. Which action will the nurse include in the plan of care?

a. Teach the patient the Credé method.
b. Instruct the patient how to self-catheterize.
c. Catheterize for residual urine after voiding.
d. Assist the patient to the toilet every 2 hours.

A

ANS: B
Because the patient’s bladder is spastic and will empty in response to overstretching of the bladder wall, the most appropriate method is to avoid incontinence by emptying the bladder at regular intervals through intermittent catheterization. Assisting the patient to the toilet will not be helpful because the bladder will not empty. The Credé method is more appropriate for a bladder that is flaccid, such as occurs with areflexic neurogenic bladder. Catheterization after voiding will not resolve the patient’s incontinence.

141
Q

When the nurse is developing a rehabilitation plan for a 30-yr-old patient with a C6 spinal cord injury, an appropriate goal is that the patient will be able to

a. drive a car with powered hand controls.
b. push a manual wheelchair on a flat surface.
c. turn and reposition independently when in bed.
d. transfer independently to and from a wheelchair.

A

ANS: B
The patient with a C6 injury will be able to use the hands to push a wheelchair on flat, smooth surfaces. Because flexion of the thumb and fingers is minimal, the patient will not be able to grasp a wheelchair during transfer, drive a car with powered hand controls, or turn independently in bed.

142
Q

A 20-yr-old patient who sustained a T2 spinal cord injury 10 days ago tells the nurse, “I want to be transferred to a hospital where the nurses know what they are doing.” Which action by the nurse is appropriate?

a. Respond that abusive language will not be tolerated.
b. Request that the patient provide input for the plan of care.
c. Perform care without responding to the patient’s comments.
d. Reassure the patient about the competence of the nursing staff.

A

ANS: B
The patient is demonstrating behaviors consistent with the anger phase of the grief process, and the nurse should allow expression of anger and seek the patient’s input into care. Expression of anger is appropriate at this stage, and should be accepted by the nurse. Reassurance about the competency of the staff will not be helpful in responding to the patient’s concerns. Ignoring the patient’s comments will increase the patient’s anger and sense of helplessness

143
Q

A 38-yr-old patient who has had a spinal cord injury returned home following a stay in a rehabilitation facility. The home care nurse notes the spouse is performing many of the activities that the patient had been managing unassisted during rehabilitation. The appropriate nursing action at this phase of rehabilitation is to

a. remind the patient about the importance of independence in daily activities.
b. tell the spouse to stop helping because the patient is able to perform activities independently.
c. develop a plan to increase the patient’s independence in consultation with the patient and the spouse.
d. recognize that it is important for the spouse to be involved in the patient’s care and encourage participation.

A

ANS: C
The best action by the nurse will be to involve all parties in developing an optimal plan of care. Because family members who will be assisting with the patient’s ongoing care need to believe their input is important, telling the spouse that the patient can perform activities independently is not the best choice. Reminding the patient about the importance of independence may not change the behaviors of the spouse. Supporting the activities of the spouse will lead to ongoing dependency by the patient.

144
Q

A patient is admitted with possible botulism poisoning after eating home-canned green beans. Which intervention ordered by the health care provider will the nurse question?

a. Encourage oral fluids to 3 L/day.
b. Document neurologic symptoms.
c. Position patient lying on the side.
d. Observe respiratory status closely.

A

ANS: A
The patient should be maintained on NPO status because neuromuscular weakness increases risk for aspiration. Side-lying position is not contraindicated. Assessment of neurologic and respiratory status is appropriate.

145
Q

Which nursing action has the highest priority for a patient who was admitted 16 hours earlier with a C5 spinal cord injury?

a. Cardiac monitoring for bradycardia
b. Assessment of respiratory rate and effort
c. Administration of low-molecular-weight heparin
d. Application of pneumatic compression devices to legs

A

ANS: B
Edema around the area of injury may lead to damage above the C4 level, so the highest priority is assessment of the patient’s respiratory function. The other actions also are appropriate for preventing deterioration or complications but are not as important as assessment of respiratory effort.

146
Q

A patient is hospitalized with new onset of Guillain-Barré syndrome. The most essential assessment for the nurse to complete is

a. determining level of consciousness.
b. checking strength of the extremities.
c. observing respiratory rate and effort.
d. monitoring the cardiac rate and rhythm.

A

ANS: C
The most serious complication of Guillain-Barré syndrome is respiratory failure, and the nurse should monitor respiratory function continuously. The other assessments will also be included in nursing care, but they are not as important as respiratory assessment.

147
Q

Before administering botulinum antitoxin to a patient in the emergency department, it is most important for the nurse to

a. obtain the patient’s temperature.
b. administer an intradermal test dose.
c. document the neurologic symptoms.
d. ask the patient about an allergy to eggs.

A

ANS: B
To assess for possible allergic reactions, an intradermal test dose of the antitoxin should be administered. Although temperature, allergy history, and symptom assessment and documentation are appropriate, these assessments will not affect the decision to administer the antitoxin.

148
Q

A patient who had a C7 spinal cord injury 1 week ago has a weak cough effort and crackles. The initial intervention by the nurse should be to

a. suction the patient’s nasopharynx.
b. notify the patient’s health care provider.
c. push upward on the epigastric area as the patient coughs.
d. encourage incentive spirometry every 2 hours during the day.

A

ANS: C
Because the cough effort is poor, the initial action should be to use assisted coughing techniques to improve the patient’s ability to mobilize secretions. The use of the spirometer may improve respiratory status, but the patient’s ability to take deep breaths is limited by the loss of intercostal muscle function. Suctioning may be needed if the patient is unable to expel secretions by coughing but should not be the nurse’s first action. The health care provider should be notified if airway clearance interventions are not effective or additional collaborative interventions are needed.

149
Q

A patient admitted with dermal ulcers who has a history of a T3 spinal cord injury tells the nurse, “I have a pounding headache and I feel sick to my stomach.” Which action should the nurse take first?

a. Check for a fecal impaction.
c. Assess the blood pressure (BP).
b. Give the prescribed antiemetic. d. Notify the health care provider.

A

ANS: C
The BP should be assessed immediately in a patient with an injury at the T6 level or higher who complains of a headache to determine if autonomic hyperreflexia is occurring. Notification of the patient’s health care provider is appropriate after the BP is obtained. Administration of an antiemetic is indicated if autonomic hyperreflexia is ruled out as the cause of the nausea. After checking the BP, the nurse may assess for a fecal impaction using lidocaine jelly to prevent further increased BP.

150
Q

A patient is being evaluated for a possible spinal cord tumor. Which finding by the nurse requires the most immediate action?

a. The patient has new-onset weakness of both legs.
b. The patient complains of chronic severe back pain.
c. The patient starts to cry and says, “I feel hopeless.”
d. The patient expresses anxiety about having surgery.

A

ANS: A
The new symptoms indicate spinal cord compression, an emergency that requires rapid treatment to avoid permanent loss of function. The other patient assessments also need nursing action but do not require intervention as rapidly as the new-onset weakness.

151
Q

Which of these nursing actions for a patient with Guillain-Barré syndrome is appropriate for the nurse to delegate to experienced unlicensed assistive personnel (UAP)?

a. Nasogastric tube feeding q4hr
b. Artificial tear administration q2hr
c. Assessment for bladder distention q2hr
d. Passive range of motion to extremities q4hr

A

ANS: D
Assisting a patient with movement is included in UAP education and scope of practice. Administration of tube feedings, administration of ordered medications, and assessment are skills requiring more education and expanded scope of practice, and the RN should perform these skills.

152
Q

Which action will the nurse take when caring for a patient who develops tetanus from injectable substance use?

a. Avoid use of sedatives.
b. Provide a quiet environment.
c. Provide range-of-motion exercises daily.
d. Check pupil reaction to light every 4 hours.

A

ANS: B
In patients with tetanus, painful seizures can be precipitated by jarring, loud noises, or bright lights, so the nurse will minimize noise and avoid shining light into the patient’s eyes. Range-of-motion exercises may also stimulate the patient and cause seizures. Although the patient has a history of injectable drug use, sedative medications will be needed to decrease spasms.

153
Q

Which action will the nurse include in the plan of care for a patient who has a cauda equina spinal cord injury?

a. Catheterize patient every 3 to 4 hours.
b. Assist patient to ambulate 4 times daily.
c. Administer medications to reduce bladder spasm.
d. Stabilize the neck when repositioning the patient.

A

ANS: A
Patients with cauda equina syndrome have areflexic bladder, and intermittent catheterization will be used for emptying the bladder. Because the bladder is flaccid, antispasmodic medications will not be used. The legs are flaccid with cauda equina syndrome, and the patient will be unable to ambulate. The head and neck will not need to be stabilized after a cauda equina injury, which affects the lumbar and sacral nerve roots.

154
Q

After change-of-shift report on the neurology unit, which patient will the nurse assess first?
a. Patient with Bell’s palsy who has herpes vesicles in front of the ear
b. Patient with botulism who is drooling and experiencing difficulty swallowing
c. Patient with neurosyphilis who has tabes dorsalis and decreased deep tendon
reflexes
d. Patient with an abscess caused by injectable drug use who needs tetanus immune
globulin

A

ANS: B
The patient’s diagnosis and difficulty swallowing indicate the nurse should rapidly assess for respiratory distress. The information about the other patients is consistent with their diagnoses and does not indicate any immediate need for assessment or intervention.

155
Q

Which finding in a patient with a spinal cord tumor requires an immediate report to the health care provider?

a. Depression about the diagnosis
b. Anxiety about scheduled surgery
c. Decreased ability to move the legs
d. Back pain that worsens with coughing

A

ANS: C
Decreasing sensation and leg movement indicates spinal cord compression, an emergency that will require rapid action (such as surgery) to prevent paralysis. The other findings will also require nursing action but are not emergencies.

156
Q

A patient with a T4 spinal cord injury asks the nurse if he will be able to be sexually active. Which initial response by the nurse is best?

a. Reflex erections frequently occur, but orgasm may not be possible.
b. Sildenafil (Viagra) is used by many patients with spinal cord injury.
c. Multiple options are available to maintain sexuality after spinal cord injury.
d. Penile injection, prostheses, or vacuum suction devices are possible options.

A

ANS: C
Although sexuality will be changed by the patient’s spinal cord injury, there are options for expression of sexuality and for fertility. The other information also is correct, but the choices will depend on the degrees of injury and the patient’s individual feelings about sexuality.

157
Q

When caring for a patient who experienced a T2 spinal cord transection 24 hours ago, which collaborative and nursing actions will the nurse include in the plan of care (select all that apply)?

a. Urinary catheter care
b. Nasogastric (NG) tube feeding
c. Continuous cardiac monitoring
d. Administration of H2 receptor blockers
e. Maintenance of a warm room temperature

A

ANS: A, C, D, E
The patient is at risk for bradycardia and poikilothermia caused by sympathetic nervous system dysfunction and should have continuous cardiac monitoring and maintenance of a relatively warm room temperature. To avoid bladder distention, a urinary retention catheter is used during this acute phase. Stress ulcers are a common complication, but can be avoided through the use of the H2 receptor blockers such as famotidine. Gastrointestinal motility is decreased initially, and NG suctioning is indicated.

158
Q

A patient with neurogenic shock after a spinal cord injury is to receive lactated Ringer’s solution 400 mL over 20 minutes. When setting the IV pump to deliver the IV fluid, the nurse will set the rate at how many milliliters per hour?

A

ANS: 1200

To administer 400 mL in 20 minutes, the nurse will need to set the pump to run at 1200 mL/hour.

159
Q

In which order will the nurse perform the following actions when caring for a patient with possible C5 spinal cord trauma who is admitted to the emergency department? (Put a comma and a space between each answer choice [A, B, C, D, E].)

a. Infuse normal saline at 150 mL/hr.
b. Monitor cardiac rhythm and blood pressure.
c. Administer O2 using a nonrebreather mask.
d. Immobilize the patient’s head, neck, and spine.
e. Transfer the patient to radiology for spinal computed tomography (CT).

A

ANS:
D, C, B, A, E
The first action should be to prevent further injury by stabilizing the patient’s spinal cord if the patient does not have penetrating trauma. Maintenance of oxygenation by administration of 100% O2 is the second priority. Because neurogenic shock is a possible complication, monitoring of heart rhythm and BP are indicated followed by infusing normal saline for volume replacement. A CT scan to determine the extent and level of injury is needed once initial assessment and stabilization are accomplished.